IBDP Maths analysis and approaches Topic: AHL 1.12 :Cartesian form HL Paper 1

Question

If \({z_1} = a + a\sqrt 3 i\) and \({z_2} = 1 – i\), where a is a real constant, express \({z_1}\) and \({z_2}\) in the form \(r\,{\text{cis}}\,\theta \), and hence find an expression for \({\left( {\frac{{{z_1}}}{{{z_2}}}} \right)^6}\) in terms of a and i.

Answer/Explanation

Markscheme

\({z_1} = 2a{\text{cis}}\left( {\frac{\pi }{3}} \right){\text{, }}{z_2} = \sqrt 2 {\text{ cis}}\left( { – \frac{\pi }{4}} \right)\)     M1     A1     A1

EITHER

\({\left( {\frac{{{z_1}}}{{{z_2}}}} \right)^6} = \frac{{{2^6}{a^6}{\text{cis(0)}}}}{{{{\sqrt 2 }^6}{\text{cis}}\left( {\frac{\pi }{2}} \right)}}\left( { = 8{a^6}{\text{cis}}\left( { – \frac{\pi }{2}} \right)} \right)\)     M1     A1     A1

OR

\({\left( {\frac{{{z_1}}}{{{z_2}}}} \right)^6} = {\left( {\frac{{2a}}{{\sqrt 2 }}{\text{cis}}\left( {\frac{{7\pi }}{{12}}} \right)} \right)^6}\)     M1     A1

\( = 8{a^6}{\text{cis}}\left( { – \frac{\pi }{2}} \right)\)     A1

THEN

\( = – 8{a^6}{\text{i}}\)     A1

Note: Accept equivalent angles, in radians or degrees. 

Accept alternate answers without cis e.g. \({\text{ = }}\frac{{8{a^6}}}{{\text{i}}}\)

[7 marks]

Question

Given that z is the complex number \(x + {\text{i}}y\) and that \(\left| {\,z\,} \right| + z = 6 – 2{\text{i}}\) , find the value of x

and the value of y .

Answer/Explanation

Markscheme

\(\sqrt {{x^2} + {y^2}} + x + y{\text{i}} = 6 – 2{\text{i}}\)     (A1)

equating real and imaginary parts     M1

\(y = – 2\)     A1

\(\sqrt {{x^2} + 4} + x = 6\)     A1

\({x^2} + 4 = {(6 – x)^2}\)     M1

\( – 32 = – 12x \Rightarrow x = \frac{8}{3}\)     A1

[6 marks]

Question

Given that \((4 – 5{\text{i}})m + 4n = 16 + 15{\text{i}}\) , where \({{\text{i}}^2} =  – 1\), find m and n if

a. m and n are real numbers;[3]

b. m and n are conjugate complex numbers.[4]
Answer/Explanation

Markscheme

a. attempt to equate real and imaginary parts     M1

equate real parts: \(4m + 4n = 16\); equate imaginary parts: \( -5m = 15\)     A1

\( \Rightarrow m = -3,{\text{ }}n = 7\)     A1

[3 marks]

b. let \(m = x + {\text{i}}y,{\text{ }}n = x – {\text{i}}y\)     M1

\( \Rightarrow (4 – 5{\text{i}})(x + {\text{i}}y) + 4(x – {\text{i}}y) = 16 + 15{\text{i}}\)

\( \Rightarrow 4x – 5{\text{i}}x + 4{\text{i}}y + 5y + 4x – 4{\text{i}}y = 16 + 15{\text{i}}\)

attempt to equate real and imaginary parts     M1

\(8x + 5y = 16,{\text{ }} -5x = 15\)     A1

\( \Rightarrow x = -3,{\text{ }}y = 8\)     A1

\(( \Rightarrow m = -3 + 8{\text{i}},{\text{ }}n = -3 – 8{\text{i}})\)

[4 marks]

 

Question

The graph of a polynomial function f of degree 4 is shown below.

Given that \({(x + {\text{i}}y)^2} = – 5 + 12{\text{i}},{\text{ }}x,{\text{ }}y \in \mathbb{R}\) . Show that

(i)     \({x^2} – {y^2} = – 5\) ;

(ii)     \(xy = 6\) .[2]

A.a.

Hence find the two square roots of \( – 5 + 12{\text{i}}\) .[5]

A.b.

For any complex number z , show that \({(z^*)^2} = ({z^2})^*\) .[3]

A.c.

Hence write down the two square roots of \( – 5 – 12{\text{i}}\) .[2]

A.d.

Explain why, of the four roots of the equation \(f(x) = 0\) , two are real and two are complex.[2]

B.a.

The curve passes through the point \(( – 1,\, – 18)\) . Find \(f(x)\) in the form

\(f(x) = (x – a)(x – b)({x^2} + cx + d),{\text{ where }}a,{\text{ }}b,{\text{ }}c,{\text{ }}d \in \mathbb{Z}\) .[5]

B.b.

Find the two complex roots of the equation \(f(x) = 0\) in Cartesian form.[2]

B.c.

Draw the four roots on the complex plane (the Argand diagram).[2]

B.d.

Express each of the four roots of the equation in the form \(r{{\text{e}}^{{\text{i}}\theta }}\) .[6]

B.e.
Answer/Explanation

Markscheme

(i)     \({(x + {\text{i}}y)^2} = – 5 + 12{\text{i}}\)

\({x^2} + 2{\text{i}}xy + {{\text{i}}^2}{y^2} = – 5 + 12{\text{i}}\)     A1

(ii)     equating real and imaginary parts     M1

\({x^2} – {y^2} = – 5\)     AG

\(xy = 6\)     AG

[2 marks]

A.a.

substituting     M1

EITHER

\({x^2} – \frac{{36}}{{{x^2}}} = – 5\)

\({x^4} + 5{x^2} – 36 = 0\)     A1

\({x^2} = 4,\, – 9\)     A1

\(x = \pm 2\) and \(y = \pm 3\)     (A1)

OR

\(\frac{{36}}{{{y^2}}} – {y^2} = – 5\)

\({y^4} – 5{y^2} – 36 = 0\)     A1

\({y^2} = 9,\, – 4\)     A1

\({y^2} = \pm 3\) and \(x = \pm 2\)     (A1) 

Note: Accept solution by inspection if completely correct.

THEN

the square roots are \((2 + 3{\text{i}})\) and \(( – 2 – 3{\text{i}})\)     A1

[5 marks]

A.b.

EITHER

consider \(z = x + {\text{i}}y\)

\(z^* = x – {\text{i}}y\)

\({(z^*)^2} = {x^2} – {y^2} – 2{\text{i}}xy\)     A1

\(({z^2}) = {x^2} – {y^2} + 2{\text{i}}xy\)     A1

\(({z^2})^* = {x^2} – {y^2} – 2{\text{i}}xy\)     A1

\({(z^*)^2} = ({z^2})^*\)     AG

OR

\(z^* = r{{\text{e}}^{ – {\text{i}}\theta }}\)

\({(z^*)^2} = {r^2}{{\text{e}}^{ – 2{\text{i}}\theta }}\)     A1

\({z^2} = {r^2}{{\text{e}}^{2{\text{i}}\theta }}\)     A1

\(({z^2})^* = {r^2}{{\text{e}}^{ – 2{\text{i}}\theta }}\)     A1

\({(z^*)^2} = ({z^2})^*\)     AG

[3 marks]

A.c.

\((2 – 3{\text{i}})\) and \(( – 2 + 3{\text{i}})\)     A1A1

[2 marks]

A.d.

the graph crosses the x-axis twice, indicating two real roots     R1

since the quartic equation has four roots and only two are real, the other two roots must be complex     R1

[2 marks]

B.a.

\(f(x) = (x + 4)(x – 2)({x^2} + cx + d)\)     A1A1

\(f(0) = – 32 \Rightarrow d = 4\)     A1

Since the curve passes through \(( – 1,\, – 18)\),

\( – 18 = 3 \times ( – 3)(5 – c)\)     M1

\(c = 3\)     A1

Hence \(f(x) = (x + 4)(x – 2)({x^2} + 3x + 4)\)

[5 marks]

B.b.

\(x = \frac{{ – 3 \pm \sqrt {9 – 16} }}{2}\)     (M1)

\( \Rightarrow x = – \frac{3}{2} \pm {\text{i}}\frac{{\sqrt 7 }}{2}\)     A1

[2 marks]

B.c.

     A1A1

 

Note: Accept points or vectors on complex plane.

Award A1 for two real roots and A1 for two complex roots.

 

[2 marks]

B.d.

real roots are \(4{{\text{e}}^{{\text{i}}\pi }}\) and \(2{{\text{e}}^{{\text{i}}0}}\)     A1A1

considering \( – \frac{3}{2} \pm {\text{i}}\frac{{\sqrt 7 }}{2}\)

\(r = \sqrt {\frac{9}{4} + \frac{7}{4}}  = 2\)     A1

finding \(\theta \) using \(\arctan \left( {\frac{{\sqrt 7 }}{3}} \right)\)     M1

\(\theta  = \arctan \left( {\frac{{\sqrt 7 }}{3}} \right) + \pi {\text{ or }}\theta  = \arctan \left( { – \frac{{\sqrt 7 }}{3}} \right) + \pi \)     A1

\( \Rightarrow z = 2{{\text{e}}^{{\text{i}}\left( {\arctan \left( {\frac{{\sqrt 7 }}{3}} \right) + \pi } \right)}}{\text{ or}} \Rightarrow z = 2{{\text{e}}^{{\text{i}}\left( {\arctan \left( {\frac{{ – \sqrt 7 }}{3}} \right) + \pi } \right)}}\)     A1 

Note: Accept arguments in the range \( – \pi {\text{ to }}\pi {\text{ or }}0{\text{ to }}2\pi \) .

Accept answers in degrees.

 

[6 marks]

B.e.

Question

Consider the complex numbers

\({z_1} = 2\sqrt 3 {\text{cis}}\frac{{3\pi }}{2}\) and \({z_2} = – 1 + \sqrt 3 {\text{i }}\) .

(i)     Write down \({z_1}\) in Cartesian form.

(ii)     Hence determine \({({z_1} + {z_2})^ * }\) in Cartesian form.[3]

a.

(i)     Write \({z_2}\) in modulus-argument form.

(ii)     Hence solve the equation \({z^3} = {z_2}\) .[6]

b.

Let \(z = r\,{\text{cis}}\theta \) , where \(r \in {\mathbb{R}^ + }\)  and \(0 \leqslant \theta  < 2\pi \) . Find all possible values of and \(\theta \) ,

(i)     if \({z^2} = {(1 + {z_2})^2}\);

(ii)     if \(z = – \frac{1}{{{z_2}}}\).[6]

c.

Find the smallest positive value of n for which \({\left( {\frac{{{z_1}}}{{{z_2}}}} \right)^n} \in {\mathbb{R}^ + }\) .[4]

d.
Answer/Explanation

Markscheme

(i)     \({z_1} = 2\sqrt 3 {\text{cis}}\frac{{3\pi }}{2} \Rightarrow {z_1} = – 2\sqrt 3 {\text{i}}\)     A1

 (ii)     \({z_1} + {z_2} = – 2\sqrt 3 {\text{i}} – 1 + \sqrt 3 {\text{i}} = – 1 – \sqrt 3 {\text{i}}\)     A1

\({({z_1} + {z_2})^ * } = – 1 + \sqrt 3 {\text{i}}\)     A1

[3 marks]

a.
(i)     \(\left| {{z_2}} \right| = 2\)
\(\tan \theta  = – \sqrt 3 \)     (M1)
\({z_2}\) lies on the second quadrant
\(\theta  = \arg {z_2} = \frac{{2\pi }}{3}\)
\({z_2} = 2{\text{cis}}\frac{{2\pi }}{3}\)     A1A1
 
(ii)     attempt to use De Moivre’s theorem     M1
\(z = \sqrt[3]{2}\,{\text{cis}}\frac{{\frac{{2\pi }}{3} + 2k\pi }}{3},{\text{ }}k = 0{\text{, 1 and 2}}\)
\(z = \sqrt[3]{2}\,{\text{cis}}\frac{{2\pi }}{9},\,\sqrt[3]{2}\,{\text{cis}}\frac{{8\pi }}{9},\,\sqrt[3]{2}\,{\text{cis}}\frac{{14\pi }}{9}\left( { = \sqrt[3]{2}\,{\text{cis}}\left( {\frac{{ – 4\pi }}{9}} \right)} \right)\)     A1A1
Note: Award A1 for modulus, A1 for arguments.
 
Note: Allow equivalent forms for z .
 
[6 marks]
b.

(i)     METHOD 1

\({z^2} = {\left( {1 – 1 + \sqrt 3 {\text{i}}} \right)^2} = – 3\left( { \Rightarrow z =  \pm \sqrt 3 {\text{i}}} \right)\)     M1

\(z = \sqrt 3 \,{\text{cis}}\frac{\pi }{2}{\text{ or }}{z_1} = \sqrt 3 \,{\text{cis}}\frac{{3\pi }}{2}\left( { = \sqrt 3 \,{\text{cis}}\left( {\frac{{ – \pi }}{2}} \right)} \right)\)     A1A1

so \(r = \sqrt 3 {\text{ and }}\theta  = \frac{\pi }{2}{\text{ or }}\theta  = \frac{{3\pi }}{2}\left( { = \frac{{ – \pi }}{2}} \right)\) 

Note: Accept \(r\,{\text{cis}}(\theta )\) form.

 

 METHOD 2

\({z^2} = {\left( {1 – 1 + \sqrt 3 {\text{i}}} \right)^2} = – 3 \Rightarrow {z^2} = 3{\text{cis}}\left( {(2n + 1)\pi } \right)\)     M1

 \({r^2} = 3 \Rightarrow r = \sqrt 3 \)     A1

\(2\theta  = (2n + 1)\pi  \Rightarrow \theta  = \frac{\pi }{2}{\text{ or }}\theta  = \frac{{3\pi }}{2}{\text{ (as }}0 \leqslant \theta  < 2\pi )\)     A1 

Note: Accept \(r\,{\text{cis}}(\theta )\) form.

 

(ii)     METHOD 1

\(z = – \frac{1}{{2{\text{cis}}\frac{{2\pi }}{3}}} \Rightarrow z = \frac{{{\text{cis}}\pi }}{{2{\text{cis}}\frac{{2\pi }}{3}}}\)     M1

\( \Rightarrow z = \frac{1}{2}{\text{cis}}\frac{\pi }{3}\)

so \(r = \frac{1}{2}{\text{ and }}\theta  = \frac{\pi }{3}\)     A1A1

METHOD 2

\({z_1} = – \frac{1}{{ – 1 + \sqrt 3 {\text{i}}}} \Rightarrow {z_1} = – \frac{{ – 1 – \sqrt 3 {\text{i}}}}{{\left( { – 1 + \sqrt 3 {\text{i}}} \right)\left( { – 1 – \sqrt 3 {\text{i}}} \right)}}\)     M1

\(z = \frac{{1 + \sqrt 3 {\text{i}}}}{4} \Rightarrow z = \frac{1}{2}{\text{cis}}\frac{\pi }{3}\)

so \(r = \frac{1}{2}{\text{ and }}\theta  = \frac{\pi }{3}\)     A1A1

[6 marks]

c.

\(\frac{{{z_1}}}{{{z_2}}} = \sqrt 3 \,{\text{cis}}\frac{{5\pi }}{6}\)     (A1)

\({\left( {\frac{{{z_1}}}{{{z_2}}}} \right)^n} = {\sqrt 3 ^n}{\text{cis}}\frac{{5n\pi }}{6}\)     A1

equating imaginary part to zero and attempting to solve     M1

obtain n = 12     A1

Note: Working which only includes the argument is valid.

[4 marks]

d.

Question

Factorize \({z^3} + 1\) into a linear and quadratic factor.[2]

a.

Let \(\gamma = \frac{{1 + {\text{i}}\sqrt 3 }}{2}\).

(i)     Show that \(\gamma \) is one of the cube roots of −1.

(ii)     Show that \({\gamma ^2} = \gamma – 1\).

(iii)     Hence find the value of \({(1 – \gamma )^6}\).[9]

b.
Answer/Explanation

Markscheme

using the factor theorem z +1 is a factor     (M1)

\({z^3} + 1 = (z + 1)({z^2} – z + 1)\)     A1

[2 marks]

a.

(i)     METHOD 1

\({z^3} = – 1 \Rightarrow {z^3} + 1 = (z + 1)({z^2} – z + 1) = 0\)     (M1)

solving \({z^2} – z + 1 = 0\)     M1

\(z = \frac{{1 \pm \sqrt {1 – 4} }}{2} = \frac{{1 \pm {\text{i}}\sqrt 3 }}{2}\)     A1

therefore one cube root of −1 is \(\gamma \)     AG

METHOD 2

\({\gamma ^2} = \left( {{{\frac{{1 + i\sqrt 3 }}{2}}^2}} \right) = \frac{{ – 1 + i\sqrt 3 }}{2}\)     M1A1

\({\gamma ^2} = \frac{{ – 1 + i\sqrt 3 }}{2} \times \frac{{1 + i\sqrt 3 }}{2} = \frac{{ – 1 – 3}}{4}\)     A1

= −1     AG

METHOD 3

\(\gamma  = \frac{{1 + i\sqrt 3 }}{2} = {e^{i\frac{\pi }{3}}}\)     M1A1

\({\gamma ^3} = {e^{i\pi }} = – 1\)     A1

 

(ii)     METHOD 1

as \(\gamma \) is a root of \({z^2} – z + 1 = 0\) then \({\gamma ^2} – \gamma + 1 = 0\)     M1R1

\(\therefore {\gamma ^2} = \gamma – 1\)     AG

Note: Award M1 for the use of \({z^2} – z + 1 = 0\) in any way.

Award R1 for a correct reasoned approach.

METHOD 2

\({\gamma ^2} = \frac{{ – 1 + i\sqrt 3 }}{2}\)     M1

\(\gamma – 1 = \frac{{1 + i\sqrt 3 }}{2} – 1 = \frac{{ – 1 + i\sqrt 3 }}{2}\)     A1

 

(iii)     METHOD 1

\({(1 – \gamma )^6} = {( – {\gamma ^2})^6}\)     (M1)

\( = {(\gamma )^{12}}\)     A1

\( = {({\gamma ^3})^4}\)     (M1)

\( = {( – 1)^4}\)

\( = 1\)     A1

METHOD 2

\({(1 – \gamma )^6}\)

\( = 1 – 6\gamma + 15{\gamma ^2} – 20{\gamma ^3} + 15{\gamma ^4} – 6{\gamma ^5} + {\gamma ^6}\)     M1A1

Note: Award M1 for attempt at binomial expansion.

 

use of any previous result e.g. \( = 1 – 6\gamma + 15{\gamma ^2} + 20 – 15\gamma  + 6{\gamma ^2} + 1\)     M1

= 1     A1

Note: As the question uses the word ‘hence’, other methods that do not use previous results are awarded no marks.

[9 marks]

b.

Question

Given that \(\frac{z}{{z + 2}} = 2 – {\text{i}}\) , \(z \in \mathbb{C}\) , find z in the form \(a + {\text{i}}b\) .

Answer/Explanation

Markscheme

METHOD 1
\(z = \left( {2 – {\text{i}}} \right)\left( {z + 2} \right)\)     M1
\( = 2z + 4 – {\text{i}}z – 2{\text{i}}\)
\(z\left( {1 – {\text{i}}} \right) = – 4 + 2{\text{i}}\)
\(z = \frac{{ – 4 + 2{\text{i}}}}{{1 – {\text{i}}}}\)     A1
\(z = \frac{{ – 4 + 2{\text{i}}}}{{1 – {\text{i}}}} \times \frac{{1 + {\text{i}}}}{{1 + {\text{i}}}}\)     M1
\( = – 3 – {\text{i}}\)     A1

METHOD 2

let \(z = a + {\text{i}}b\)
\(\frac{{a + {\text{i}}b}}{{a + {\text{i}}b + 2}} = 2 – {\text{i}}\)     M1
\(a + {\text{i}}b = \left( {2 – i} \right)\left( {\left( {a + 2} \right) + {\text{i}}b} \right)\)
\(a + {\text{i}}b = 2\left( {a + 2} \right) + 2b{\text{i}} – {\text{i}}\left( {a + 2} \right) + b\)
\(a + {\text{i}}b = 2a + b + 4 + \left( {2b – a – 2} \right){\text{i}}\)
attempt to equate real and imaginary parts     M1
\(a = 2a + b + 4\left( { \Rightarrow a + b + 4 = 0} \right)\)
and \(b = 2b – a – 2\left( { \Rightarrow – a + b – 2 = 0} \right)\)     A1

Note: Award Al for two correct equations.

 \(b = – 1\); \(a = – 3\)     A1

\(z = – 3 – {\text{i}}\)

[4 marks]

Question

Write down the expansion of \({\left( {\cos \theta + {\text{i}}\sin \theta } \right)^3}\) in the form \(a + {\text{i}}b\) , where \(a\) and \(b\) are in terms of \({\sin \theta }\) and \({\cos \theta }\) .[2]

a.

Hence show that \(\cos 3\theta = 4{\cos ^3}\theta – 3\cos \theta \) .[3]

b.

Similarly show that \(\cos 5\theta = 16{\cos ^5}\theta – 20{\cos ^3}\theta + 5\cos \theta \) .[3]

c.

Hence solve the equation \(\cos 5\theta + \cos 3\theta + \cos \theta = 0\) , where \(\theta \in \left[ { – \frac{\pi }{2},\frac{\pi }{2}} \right]\) .[6]

d.

By considering the solutions of the equation \(\cos 5\theta = 0\) , show that \(\cos \frac{\pi }{{10}} = \sqrt {\frac{{5 + \sqrt 5 }}{8}} \) and state the value of \(\cos \frac{{7\pi }}{{10}}\).[8]

e.
Answer/Explanation

Markscheme

\({\left( {\cos \theta + {\text{i}}\sin \theta } \right)^3} = {\cos ^3}\theta + 3{\cos ^2}\theta \left( {{\text{i}}\sin \theta } \right) + 3\cos \theta {\left( {{\text{i}}\sin \theta } \right)^2} + {\left( {{\text{i}}\sin \theta } \right)^3}\)     (M1)

\( = {\cos ^3}\theta – 3\cos \theta {\sin ^2}\theta + {\text{i}}\left( {3{{\cos }^2}\theta \sin \theta – {{\sin }^3}\theta } \right)\)     A1

[2 marks]

a.

from De Moivre’s theorem

\({\left( {\cos \theta + {\text{i}}\sin \theta } \right)^3} = \cos 3\theta + {\text{i}}\sin 3\theta \)     (M1)

\(\cos 3\theta + {\text{i}}\sin 3\theta = \left( {{{\cos }^3}\theta – 3\cos \theta {{\sin }^2}\theta } \right) + {\text{i}}\left( {3{{\cos }^2}\theta \sin \theta – {{\sin }^3}\theta } \right)\)

equating real parts     M1

\(\cos 3\theta = {\cos ^3}\theta – 3\cos \theta {\sin ^2}\theta \)

\( = {\cos ^3}\theta – 3\cos \theta \left( {1 – {{\cos }^2}\theta } \right)\)     A1

\( = {\cos ^3}\theta – 3\cos \theta + 3{\cos ^3}\theta \)

\( = 4{\cos ^3}\theta – 3\cos \theta \)     AG

Note: Do not award marks if part (a) is not used.

 [3 marks]

b.

\({\left( {\cos \theta + {\text{i}}\sin \theta } \right)^5} = \)

\({\cos ^5}\theta + 5{\cos ^4}\theta \left( {{\text{i}}\sin \theta } \right) + 10{\cos ^3}\theta {\left( {{\text{i}}\sin \theta } \right)^2} + 10{\cos ^2}\theta {\left( {{\text{i}}\sin \theta } \right)^3} + 5\cos \theta {\left( {{\text{i}}\sin \theta } \right)^4} + {\left( {{\text{i}}\sin \theta } \right)^5}\)     (A1)

from De Moivre’s theorem

\(\cos 5\theta = {\cos ^5}\theta – 10{\cos ^3}\theta {\sin ^2}\theta  + 5\cos \theta {\sin ^4}\theta \)     M1

\( = {\cos ^5}\theta – 10{\cos ^3}\theta \left( {1 – {{\cos }^2}\theta } \right) + 5\cos \theta {\left( {1 – {{\cos }^2}\theta } \right)^2}\)     A1

\( = {\cos ^5}\theta – 10{\cos ^3}\theta + 10{\cos ^5}\theta + 5\cos \theta – 10{\cos ^3}\theta + 5{\cos ^5}\theta \)

\(\therefore \cos 5\theta = 16{\cos ^5}\theta – 20{\cos ^3}\theta + 5\cos \theta \)     AG

Note: If compound angles used in (b) and (c), then marks can be allocated in (c) only.

 [3 marks]

c.

\(\cos 5\theta + \cos 3\theta + \cos \theta \)

\( = \left( {16{{\cos }^5}\theta – 20{{\cos }^3}\theta + 5\cos \theta } \right) + \left( {4{{\cos }^3}\theta – 3\cos \theta } \right) + \cos \theta = 0\)     M1

\(16{\cos ^5}\theta – 16{\cos ^3}\theta + 3\cos \theta = 0\)     A1

\(\cos \theta \left( {16{{\cos }^4}\theta – 16{{\cos }^2}\theta + 3} \right) = 0\)

\(\cos \theta \left( {4{{\cos }^2}\theta – 3} \right)\left( {4{{\cos }^2}\theta – 1} \right) = 0\)     A1

\(\therefore \cos \theta = 0\); \( \pm \frac{{\sqrt 3 }}{2}\); \( \pm \frac{1}{2}\)     A1

\(\therefore \theta = \pm \frac{\pi }{6}\); \(\pm \frac{\pi }{3}\); \( \pm \frac{\pi }{2}\)     A2

[6 marks]

d.

\(\cos 5\theta = 0\)

\(5\theta = …\frac{\pi }{2}\); \(\left( {\frac{{3\pi }}{2};\frac{{5\pi }}{2}} \right)\); \(\frac{{7\pi }}{2}\); \(…\)     (M1)

\(\theta = …\frac{\pi }{{10}}\); \(\left( {\frac{{3\pi }}{{10}};\frac{{5\pi }}{{10}}} \right)\); \(\frac{{7\pi }}{10}\); \(…\)     (M1)

Note: These marks can be awarded for verifications later in the question.

 now consider \(16{\cos ^5}\theta – 20{\cos ^3}\theta + 5\cos \theta = 0\)     M1

\(\cos \theta \left( {16{{\cos }^4}\theta – 20{{\cos }^2}\theta + 5} \right) = 0\)

\({\cos ^2}\theta = \frac{{20 \pm \sqrt {400 – 4\left( {16} \right)\left( 5 \right)} }}{{32}}\); \(\cos \theta = 0\)     A1

\(\cos \theta = \pm \sqrt {\frac{{20 \pm \sqrt {400 – 4\left( {16} \right)\left( 5 \right)} }}{{32}}} \)

\(\cos \frac{\pi }{{10}} = \sqrt {\frac{{20 + \sqrt {400 – 4\left( {16} \right)\left( 5 \right)} }}{{32}}} \) since max value of cosine \( \Rightarrow \) angle closest to zero     R1

\(\cos \frac{\pi }{{10}} = \sqrt {\frac{{4.5 + 4\sqrt {25 – 4\left( 5 \right)} }}{{4.8}}}  = \sqrt {\frac{{5 + \sqrt 5 }}{8}} \)     A1

\(\cos \frac{{7\pi }}{{10}} = – \sqrt {\frac{{5 – \sqrt 5 }}{8}} \)     A1A1

[8 marks]

e.

Question

Solve the equation \({z^3} = 8{\text{i}},{\text{ }}z \in \mathbb{C}\) giving your answers in the form \(z = r(\cos \theta  + {\text{i}}\sin \theta )\) and in the form \(z = a + b{\text{i}}\) where \(a,{\text{ }}b \in \mathbb{R}\).[6]

a.

Consider the complex numbers \({z_1} = 1 + {\text{i}}\) and \({z_2} = 2\left( {\cos \left( {\frac{\pi }{2}} \right) + {\text{i}}\sin \left( {\frac{\pi }{6}} \right)} \right)\).

(i)     Write \({z_1}\) in the form \(r(\cos \theta  + {\text{i}}\sin \theta )\).

(ii)     Calculate \({z_1}{z_2}\) and write in the form \(z = a + b{\text{i}}\) where \(a,{\text{ }}b \in \mathbb{R}\).

(iii)     Hence find the value of \(\tan \frac{{5\pi }}{{12}}\) in the form \(c + d\sqrt 3 \), where \(c,{\text{ }}d \in \mathbb{Z}\).

(iv)     Find the smallest value \(p > 0\) such that \({({z_2})^p}\) is a positive real number.[11]

b.
Answer/Explanation

Markscheme

Note:     Accept answers and working in degrees, throughout.

\({z^3} = 8\left( {\cos \left( {\frac{\pi }{2} + 2\pi k} \right) + {\text{i}}\sin \left( {\frac{\pi }{2} + 2\pi k} \right)} \right)\)     (A1)

attempt the use of De Moivre’s Theorem in reverse     M1

\(z = 2\left( {\cos \left( {\frac{\pi }{6}} \right) + {\text{i}}\sin \left( {\frac{\pi }{6}} \right)} \right);{\text{ }}2\left( {\cos \left( {\frac{{5\pi }}{6}} \right) + {\text{i}}\sin \left( {\frac{{5\pi }}{6}} \right)} \right);\)

\(2\left( {\cos \left( {\frac{{9\pi }}{6}} \right) + {\text{i}}\sin \left( {\frac{{9\pi }}{6}} \right)} \right)\)     A2

Note:     Accept cis form.

\(z =  \pm \sqrt 3  + {\text{i}},{\text{ }} – 2{\text{i}}\)     A2

Note:     Award A1 for two correct solutions in each of the two lines above.

[6 marks]

a.

Note:     Accept answers and working in degrees, throughout.

(i)     \({z_1} = \sqrt 2 \left( {\cos \left( {\frac{\pi }{4}} \right) + {\text{i}}\sin \left( {\frac{\pi }{4}} \right)} \right)\)     A1A1

(ii)     \(\left( {{z_2} = \left( {\sqrt 3  + {\text{i}}} \right)} \right)\)

\({z_1}{z_2} = (1 + {\text{i}})\left( {\sqrt 3  + {\text{i}}} \right)\)     M1

\( = \left( {\sqrt 3  – 1} \right) + {\text{i}}\left( {1 + \sqrt 3 } \right)\)     A1

(iii)     \({z_1}{z_2} = 2\sqrt 2 \left( {\cos \left( {\frac{\pi }{6} + \frac{\pi }{4}} \right) + {\text{i}}\sin \left( {\frac{\pi }{6} + \frac{\pi }{4}} \right)} \right)\)     M1A1

Note:     Interpret “hence” as “hence or otherwise”.

\(\tan \frac{{5\pi }}{{12}} = \frac{{\sqrt 3  + 1}}{{\sqrt 3  – 1}}\)     A1

\( = 2 + \sqrt 3 \)     M1A1

Note:     Award final M1 for an attempt to rationalise the fraction.

(iv)     \({z_2}^p = {2^p}\left( {{\text{cis}}\left( {\frac{{p\pi }}{6}} \right)} \right)\)     (M1)

\({z_2}^p\) is a positive real number when \(p = 12\)     A1

Note:     Accept a solution based on part (a).

[11 marks]

Total [17 marks]

b.
Scroll to Top